Select the correct answer. Which term is a term in this expression? -3x - 7(x+4) OA x + 4 O B. -7 Ос. -3x OD. -3​

Answers

Answer 1

Answer:

D. -3x

Step-by-step explanation:

Terms have the form k or kx^n, where k is a number, x is a variable, and n is a whole number.

-3x follows the above rule, so -3x is a term.

Answer 2

Answer:

D

Step-by-step explanation:

Hi...


Related Questions

square root of 2025 by prime factorisation method​
Please explain step by step

Answers

Answer:

45

Step-by-step explanation:2025....use the smallest prime number to divide the number 2025 till you get to a point were it can't be divided again . Group each prime number with a partner of its value like this :

5×5×3×3×3×3........ then use bracket to separate them like this (5×5)(3×3)(3×3)........ then take a number from each 5×3×3 then multiply to get 45

the sum of the arithmetic series: a1 = 9, a3 = 17, s5 =? ​

Answers

[tex]\huge\underline\bold\blue{ƛƝƧƜЄƦ}[/tex]

Given

[tex]\blue\star[/tex]a1 = 9. eq1

[tex]\blue\star[/tex]a2 = 17. eq2

Firstly we have to find d

[tex]\blue\star[/tex]d = a2 -a1

we dont have a2 so we can do this by other method

split a3 into a+2d

Now add equation 1 and 2

[tex]\blue\star[/tex] a+a+2d = 17 +9

[tex]\blue\star[/tex] 18 + 2d = 26. [we have a = 9]

[tex]\blue\star[/tex] 2d = 8

[tex]\blue\star[/tex] d = 4

Now we have to find a5

[tex]\blue\star[/tex] a5 = a+(n-1)d

[tex]\blue\star[/tex] a5 = 9 + (5-1)4

[tex]\blue\star[/tex] a5 = 25

now we can easily find s5

[tex]\blue\star[/tex]s5 = n\2(a+l)

[tex]\blue\star[/tex] s5 =5\2(9+25)

[tex]\blue\star[/tex] s5 = 85

[tex]\huge\boxed{S5 = 85}[/tex]

Check

s5 = a1 + a2 + a3 +a4 +a5

s5 = 9 +13+17+21+25= 85

Hope it helps

The sum of the arithmetic sequence [tex]S_5=85[/tex]

Explanation :

Arithmetic series, a1 = 9, a3 = 17

We need to find out sum of 5 terms

The formula to find sum of arithmetic series is

[tex]S_n=\frac{n}{2} (2a+(n-1)d)[/tex]

where 'a' is the first term

'd' is the common difference and n is the number of terms

a1=9,, first term is 9

formula for nth term is [tex]a_n=a_1+(n-1)d[/tex]

a3=17, it means third term is 17

lets find out 'd' using third term

[tex]a_3=a_1+(3-1)d\\17=9+2d\\17-9=2d\\8=2d\\d=4[/tex]

So value of d=4

Now use sum formula to find out S5

[tex]S_n=\frac{n}{2} (2a+(n-1)d)\\n=5, d=4, a_1=9\\\\S_5=\frac{5}{2} (2(9)+(5-1)4)\\S_5=\frac{5}{2}\cdot \:34\\S_5=85[/tex]

Learn more : brainly.com/question/12733379

What is a Metric prefix?

Answers

Answer:

   

Step-by-step explanation:

A metric prefix is a unit prefix that precedes a basic unit of measure to indicate a multiple or submultiple of the unit. All metric prefixes used today are decadic. Each prefix has a unique symbol that is prepended to any unit symbol.

This is the definition

A store charges $120 for a bicycle. The store is now going out of business and is
selling all of the bicycles at a 30% discount. What is the sale price of the bicycle?

Answers

Answer:

$84

Step-by-step explanation:

So, if the store is selling the bicycles at a 30% discount, that means that the bike is at 70% of it's original value. SO we can do 120*.7=84.

Comelia and Christopher are arguing about Real Number (R) Sets. Comelia says that all Whole (W) numbers are Rational (Q) and Christopher
says that all Rational (Q) numbers are Whole (W). Who is correct and explain why?
Unread
Search entries or author
EL
1
Edit Insert Format Tools
Table
<
>
2
Paragraph
B.
12pt
I U Avert²v
I

Answers

Answer: Comelia is correct

====================================================

Explanation:

We're told that "Christopher  says that all Rational (Q) numbers are Whole (W)", which makes Christopher not correct. Some rational numbers are whole numbers. For instance, the number 7 = 7/1 is rational and it's a whole number as well.

However something like 1/2 is rational, but it's not a whole number. A whole number doesn't have any fractional or decimal part to it. It can be thought of the number of something.

Comelia is correct because all whole numbers are rational. If x is some whole number, then x = x/1 is rational as well. Replace x with any whole number you want. Her statement does not work in reverse as shown above.

When drawing a Venn diagram, the circle for "whole numbers" will be entirely inside the circle for "rational numbers", and not the other way around.

Write the phrase as an expression
the quotient of 22 and a number a

Answers

Answer:

22/a

Step-by-step explanation:

The quotient of 22 and a number = 22/a

I am in class I really need help the teacher is gonna call on me help please

Answers

Answer:

5/10 or 1/2

Step-by-step explanation:

5-4 is 1, and 1 squared is still 1.

the cube root of 27 is 3

7 to the 0 power is 1

1 + 3 + 1 is 5

5 over 10 is 1/2

Answer:

1/2

Step-by-step explanation:

(5-4)²+∛27+[tex]7^{0}[/tex]

1²+∛27+[tex]7^{0}[/tex]

1+∛27+[tex]7^{0}[/tex]

1+3+[tex]7^{0}[/tex]

1+3+1

4+1

5/10=1/2

5 + 2x < 2x + 6
what is x?

Answers

Answer:

x = infinite amount of solutions

Step-by-step explanation:

Step 1: Write out inequality

5 + 2x < 2x + 6

Step 2: Solve

5 < 6

Since we can see that 6 is indeed bigger than 5, it makes the inequality true. If you plug in any number x, it will still be true. Therefore, we have an infinite amount of solutions,

3
Exress 10° in standard form
1
10
100

Answers

Answer:

1

Step-by-step explanation:

In general [tex]a^{0}[/tex] = 1 ( where a is a real number ), thus

[tex]10^{0}[/tex] = 1

Can y’all help me what do I do here please I’ll give you brainliest

Answers

Answer:

1, 2, -3, 51

Step-by-step explanation:

0 ÷ 2 = 0

0 + 1 = 1

2 ÷ 2 = 1

1 + 1 = 2

-8 ÷ 2 = -4

-4 + 1 = -3

100 ÷ 2 = 50

50 + 1 = 51


Find the arc length of the semicircle.
Either enter an exact answer in terms of it or use 3.14 for IT and enter your answer as a decimal.

Answers

A semicircle is a half of a circle.

Find the circumference of a full circle then divide in half.

Circumference = pi x r x 2

Circumference = 3.14 x 9 x 2 = 56.52

Semicircle = 56.52/2 = 28.26

Which represents a function? y x y -3 -5 10 {(-8, -2), (-4,1). (0,-2), (2, 3), (4,-4)} -2 -3 5 {(-12,4),(-6, 10), (-4, 15).(-8, 18). (-12, 24) -2 -3 4 0 -1 0 0 0 0 5 -10 1 -1​

Answers

The 2nd one , because your x value should only repeat one time , and it follows those standards.

The tabular data shown in the image attached does not represent a function.

What is function?

A function is a relation between a dependent and independent variable. We can write the examples of function as -

y = f(x) = ax + b

y = f(x, y, z) = ax + by + cz

Given is to find which of the given tables represent a function.

For a table to represent a function, for every unique value of {x}, there should be only one possible value of {y}. This means that if for a function f(x), at {x} = 1, there exists two or more values of {y}, then it is not a function. From the image, it can be seen that the relations given in tables does not represent a function. This is because -

Table {1} -

for {x} = - 3, there exists two values of y

Table {3} -

for {x} = 0, there exists two values of y

Therefore, the tabular data shown in the image attached does not represent a function.

To solve more questions on functions, visit the link below-

https://brainly.com/question/30194223

#SPJ7

win) = 2n + 1; Find w(6)

Answers

Answer:

13

Step-by-step explanation:

w(n) = 2n + 1;

Let n = 6

w(6)= 2*6 +1

      = 12 +1

      = 13

Answer:

[tex]\Huge \boxed{w(6)=13}[/tex]

[tex]\rule[225]{225}{2}[/tex]

Step-by-step explanation:

[tex]w(n)=2n+1[/tex]

Replace n with 6.

[tex]w(6)=2(6)+1[/tex]

Evaluating:

[tex]w(6)=12+1[/tex]

[tex]w(6)=13[/tex]

[tex]\rule[225]{225}{2}[/tex]

Can someone help me with #s 16 18 19 20 I WILL GIVE BRAINLIEST

Answers

Answers & Step-by-step explanation:

PEMDAS (Parenthesis, exponents, multiplication, division, addition, subtraction)

Now let's solve your problems!

___________________________________________________16.

First, we see if there are any parentheses. Since there are no parentheses, we will evaluate the exponent 3² first.

After this, our problem now looks like this:

[tex]\frac{41 - 9}{6 * 3 -26}[/tex]

Next, since multiplication (not subtraction) is next on the list, we will evaluate 6 × 3, which equals 18.

After that, we will subtract:

41 - 9 = 32

18 - 26 = -8

We now have 32/-8 or 4/-1 or simply -4.

The answer is: -4___________________________________________________17.

With this, one, note the parentheses. This means that we will do these operations first.

(21 - 4²) = 5

(3 + 1) = 4

The problem now looks like this:

[tex]\frac{5 * 4}{8^{2}-44}[/tex]

Next, we do the exponents:

8² = 64

After that, we do the multiplication of 5 x 4, which equals 20.

We now have 20/64 - 44, which simplifies to 20/20 or 1/1 or 1.

The answer is: 1___________________________________________________18.

Again, we see parentheses, so that means we need to do these operations first.

(6² - 20) = 16

We now have:

48 ÷ 16 + 5

48/16 = 3 and 3 + 5 = 8

The answer is: 8___________________________________________________19.

Here, we see an exponent 3³, so we'll need to evaluate this first.

3³ = 27

We now have:

4 × 27 ÷ 18 + 14

4 × 27 = 108

108/18 = 6

And finally:

6 + 14 = 20

The answer is: 20___________________________________________________20.

Here we again see the parentheses, which means we need to evaluate the subtraction first.

3 - 8 = -5

We now have:

11 + 2 (-5)

2 × -5 = -10

11 + -10 (same as 11 - 10) = 1

The answer is: 1___________________________________________________I AM ALWAYS HAPPY TO HELP :)

g ( h + 3/5 ) = 11 solve for h

Answers

Answer:

h = 11/g - 3/5

Step-by-step explanation:

[tex]g ( h + \frac{3}{5} ) = 11 \\\\Divide\:both\:sides\:of\:the\:equation\:by\:g \:;\\\frac{g(h+\frac{3}{5}) }{g} = \frac{11}{g} \\\\h + \frac{3}{5} = \frac{11}{g} \\\\Move\: 3/5\: to\: the\: right\:and\:change\:its\:sign\\h = \frac{11}{g} -\frac{3}{5}[/tex]

Phil weighs 120 pounds and is gaining ten pounds each month. Phil weighs 150 pounds and is gaining 4 pounds each month. How many months, m, will it take for Bill to weigh the same as Phil?

Answers

Answer:

5 months

Step-by-step explanation:

I'm assuming there was a typo and said that Phil weighed both amounts so I'm assuming that the second amount is Bill's weight

We can use LCM for this

So lets make a list for both of them

Phil:120, 130, 140, 150, 160, 170, 180, 190, 200

Bill: 150, 154, 158, 162, 166, 170, 174

As you can see here, after 5 months, they will have the same weight.

We can check our work to.

lets do 120+(10*5)=170

and 150+(4*5)=170

I hope this makes sense

Answer:

5 months

Step-by-step explanation:

lets chart it like i did as you can see they both started at 120 and 150 then 5 month later there both 170

bill              phil

start . 120      150

1m.130       154

2m.140       158

3m.150       162

4m.160       166

5m.170        170

Add the polynomials (7x3−2x2−12)+(−3x3−8x2+10x)

Answers

Answer:

See below

Step-by-step explanation:

● (7x^3 - 2x^2 -12) + (-3x^3 - 8x^2 +10x)

● 7x^3 - 2x^2 - 12- 3x^3 - 8x^2 + 10x

Combine like terms

● 7x^3 - 3x^3 -2x^2 - 8x^2 -12 + 10x

● 4x^3 - 10x^3 +10x -12

Answer:

Addition:::::4x^3-10x^2+10x-12

Is y=x^2-6 a linear function and why

Answers

Answer:

No.

Step-by-step explanation:

No, y=x²-6 is not a linear function.

This is because all linear functions have the highest exponent of 1. Anything equation that has a variable raised to a power other than 1 is not linear.

The equation provided as the highest exponent of 2. Thus, it's not linear.

From the provided graph, we can also see that this is quadratic instead of linear. The graph is a parabola instead of a line.

Answer:

NO! y=x^2-6 is nonlinear

Step-by-step explanation:

A great teacher once told me the easiest way to tell if an equation is linear or not, by just looking at it is, if you see that the equation contains an exponent then you should automatically know that it is nonlinear. Therefore the equation y=x^2-6 is nonlinear!!!

Determine the domain of the function (fog)(x) where f(x)=3x-1/x-4 and g(x)=x+1/x

Answers

Answer: (-∞,-1) ∪ (0,+∞)

Step-by-step explanation: The representation fog(x) is a representation of composite function, meaning one depends on the other.

In this case, fog(x) means:

fog(x) = f(g(x))

fog(x) = [tex]3(x+\frac{1}{x} )-\frac{1}{x+\frac{1}{x} } -4[/tex]

[tex]fog(x)=3x+\frac{3}{x} -\frac{1}{\frac{x^{2}+x}{x} } -4[/tex]

[tex]fog(x)=3x+\frac{3}{x} -\frac{x}{x^{2}+x} -4[/tex]

[tex]fog(x)=\frac{3x^{2}(x^{2}+x)+3(x^{2}+x)-x-4x(x^{2}+x)}{x(x^{2}+x)}[/tex]

[tex]fog(x)=\frac{3x^{4}+3x^{3}+3x^{2}+3x-x-4x^{3}+4x^{2}}{x(x^{2}+x)}[/tex]

[tex]fog(x)=\frac{3x^{4}-x^{3}-x^{2}+2x}{x(x^{2}+x)}[/tex]

This is the function fog(x).

The domain of a function is all the values the independent variable can assume.

For fog(x), denominator can be zero, so:

[tex]x(x^{2}+x) \neq 0[/tex]

If x = 0, the function doesn't exist.

[tex]x^{2}+x \neq0[/tex]

[tex]x(x+1) \neq0[/tex]

[tex]x+1\neq0[/tex]

[tex]x\neq-1[/tex]

Therefore, the domain of this function is: -∞ < -1 or x > 0

Jonah starts jogging at a constant speed of 6 miles per hour, Five minutes later, Sory jogging from the sa me place along the same trail. Sory jogs at a constant speed of 7.5 miles per hour. How far apart are they after Sory jogs for 12 minutes?​

Answers

Answer:

0.2 miles

Step-by-step explanation:

Given that Jonah starts jogging at a constant speed of 6 miles per hour, Five minutes later, Jonah must have covered a distance of;

Distance = speed × time

Distance = 6 × 5/60

Distance = 0.5 miles

Distance covered in 12 minute will be

Distance = 6 × 12/60

Distance = 1.2 miles

Total distance covered by Jonah will be 0.5 + 1.2 = 1.7 miles

Sory jogging from the same place along the same trail. Sory jogs at a constant speed of 7.5 miles per hour.

If Sory jogs for 12 minute, then sory distance covered will be

Distance = speed × time

Distance = 7.5 × 12/60

Distance = 1.5

The distance between Jonah and Sory will be:

1.7 - 1.5 = 0.2 miles

Therefore, they are 0.2 miles apart after 12 minutes

PLEASE HELP! 15 POINTS Order the expressions from greatest to least coefficient of x. Place the expression with the greatest coefficient of x at the top. 12+4x, 13x, 15-x, 14-7x----------My answer: 13x, 14-7x, 12+4x, 15-x

Answers

Answer:

you're right; 13x, 14 - 7x, 12 + 4x, 15 - x

13, 7, 4, 1

there is a number that if you multiple it by 5 first, and then add 6 to the result, you get 91. what’s the number?

Answers

Answer:

Step-by-step explanation:

Do 91-6 which equals 85. Then do 85/5 which equals 17

answer: 17

Answer:

17

Step-by-step explanation:

So first we write an equation

5x+6=91

subtract 6 from both sides

5x=85

divide by 5

so x=17

(plz give me brainliest)

Which is a common denominator of 1/5 and 1/10

Answers

Answer:

1

Step-by-step explanation:

i'm in middle school answering a question that can be solved

Answer:

10

Step-by-step explanation:

take LCM of 5 and 10 to find the common denominator

Solve -17 + n over 5 = 33 please help :(

Answers

the answer is n=182 !
Omg I was stuck on that too thank you so much

A passenger car weighs 4.5 times 10 Superscript 3 units. Which unit of measure was used?




Answers

20? I’m not for sure maybe look in the book? That’s the answer I got and it was right

Answer:

pounds

Source:

Trust me bro

1) How would you solve consecutive integers? 2) What happen to the equations when you solve consecutive EVEN integers? 3) What about consecutive ODD integers? ​

Answers

Answer:

(a) make use of average values; (b) use a variable (n) and consecutive values: n+1, n+2, and so onEven integers differ by 2 instead of 1, so they would be represented by n, n+2, n+4, and so on.Odd integers also differ by 2, so they would have the same representation.

Step-by-step explanation:

1) If the sum of consecutive integers is given, I usually solve the problem by considering the average of those integers: the sum divided by the number of integers. (See https://brainly.com/question/17307802 for an example.)

If some other relation is given then quite often a variable is assigned to one of the integers, often the smallest. If they are consecutive, then the others may be n+1, n+2, n+3 and so on.

__

2) If they are consecutive even or odd, then they differ by 2 instead of 1: n, n+2, n+4, and so on.

__

3) That difference of 2 is the same whether the integers are consecutive even or consecutive odd. The representation used does not change.

_____

Additional comment

In some instances, you need to ensure that the integers are even or odd, so you might use a representation of 2n for an even integer and 2n+1 for an odd integer. This is rarely an issue in consecutive integer problems.

Felicity accidentally overdrew her bank account by $24.65. She then deposited a birthday check from her grandma for $25.00, paid her phone bill of $46.75, and deposited $32.80 she found in her wallet. What was the final balance of her account? Plz Answer!

Answers

Answer:

It would be  -13.6

Step-by-step explanation:

I used my calculator for this,  

-24.65 +25 - 46.75 + 32.80 = -13.6

i would love if you could answer very soon

Which ordered pair is a solution to the system of inequalities?
S y> -2
x + y < 4

Answers

Answer:

4

Step-by-step explanation: you have t do y plus 10 and then add 4  minus -2

What is the equation of a line that passes through (4, 3) and has a slope of 2?

A. y=2x – 11
B. y=2x-5
C. y=2x - 1
D. y=2x - 7

Answers

i would have to say

letter C. y=2x-1

Find the smallest number which when divided by 12,15,18&27 leaves as remainder 8,11,14,23 respectively.

Answers

Answer:

[tex] \boxed{ \bold{ \huge{ \boxed{ \sf{536}}}}}[/tex]

Step-by-step explanation:

Solution :

Here, 12 - 8 = 4

15 - 11 = 4

18 - 14 = 4

27 - 23 = 4

Thus, every divisor is greater than its remainder by 4. So, the required smallest number is the difference of the L.C.M of the given number and 4

Finding the L.C.M

First of find the prime factors of each numbers

12 = 2 × 2 × 3

15 = 3 × 5

18 = 3 × 3 × 3

27 = 3 × 3 × 3

Take out the common prime factors : 3 , 3 and 3

Also take out the other remaining prime factors : 2 , 2 and 5

Now, Multiply those all prime factors and obtain L.C.M

L.C.M = Common factors × Remaining factors

= 3 × 3 × 3 × 2 × 2 × 5

= 540

L.C.M of 12 , 15 , 18 and 27 = 540

So, The required smallest number = 540 - 4

= 536

Hope I helped!

Best regards!!

Other Questions
A = (b1 + b2)h Solve for b1 Glenn bought 3 pounds of tomatoes. He used bles of them to make sauce. Make an equation that shows the number of pounds of tomatoes Glenn used for the sauce. Move numbers and symbols to the blanks to complete the equation. If there is no whole number in the answer, put a 0 in the first box after the equals sign. Please, someone help!The order of magnitude of the distance to size ratio of the Sun- Nearest Star system is about ______ times that of the Sun-Earth system. References:The distance to size ratio of the Sun- Nearest Star system is 100,000,000The distance to size ratio of the Sun-Earth system is 10,000Hint you have to divide them, however, the answer is not 10,000Please help! I will mark the brainliest! -3x + 5 = 2x + 10 what is the answer HELP WILL GIVE POINTS NEED HELP ASAP Select the correct location on the map.In which region were sheep and cattle first domesticated? Weights and heights of turkeys tend to be correlated. For a population of turkeys at a farm, this correlation is found to be 0.64. The average weight is 17 pounds, SD is 5 pounds. The average height is 28 inches and the SD is 8 inches. Weight and height both roughly follow the normal curve. For each part below, answer the question or if not possible, indicate why not. A turkey at the farm which weighs more than 90% of all the turkeys is predicted to be taller than % of them. The average height for turkeys at the 90th percentile for weight is Of the turkeys at the 90th percentile for weight, roughly what percent would you estimate to be taller than 28 inches? An issue of common stock has just paid a dividend of $2.00. Its growth rate is equal to 4%. If the required rate of return is 7%, what is its current price Consider the function represented by 9x+3y=12 with x as the independent variable. How can this function be written using function notation? f(y)=1/3v+4/3 f(x)=-3x+4 f(x)-1/3x+4/3 f(y)=-3y+4 Which of these people felt African Americans should be able to serve in the Continental Army as a way to emancipation (freedom )?1. Lord dunmore2. George Washington 3. John adams 4. Alexander Hamiliton Six times a number is at least four less than eight times the number. If x represents the number, which will help find the number? Chang is on his way home in his car. He had driven 36 so far, which is three-fourths of the way home. What is the total length of his drive. The capital budgeting decision process can be described as how a firm's day-to-day financial matters should be managed. how a firm should finance its assets. which productive assets a firm should purchase. all of the above. A 1.0 mole pure sample of molten tin is dissolved in a 5.0 mole pure sample of molten copper. The solution is set aside to cool and solidify. The atomic radius of tin is 140 pm and the atomic radius of copper is 128 pm. a. Identify the type of alloy that is formed. Justify your answer. b. Identify the solvent in this solution. Justify your answer. the linen department had net sales of $80,000. there was a 2% loss, and the gross margin was 46%. determine the operating expenses of the department and express the result in dollars and as a percentage twice the sum of two numbers is 36 If one of the numbers is 10 find the other number PLEASE HELP! WILL GIVE BRAINLIEST! Are traits controlled by dominant alleles more common than traits controlled by recessive alleles? Please explain Is the expression 3x^2-5x^3 a Polynomial? If yes, what is the degree? The human body requires a minimum of 100mg per day of calcium.O TrueO FalseWILL MARK AS BRAINLISET THE FASTER YOU REPLY :) What is (2x + 3)??(2x + 3)?1x2 + As a fraction what is 5/8+3/5